LSAT考试全真题二SECTION1
来源:优易学  2010-1-13 18:16:08   【优易学:中国教育考试门户网】   资料下载   外语书店

 

Questions 19-24

Within a tennis league each of five teams occupies one of five positions, numbered 1 through 5 in order of rank, with number 1 as the highest position. The teams are initially in the order R, J, S, M, L, with R in position 1. Teams change positions only when a lower-positioned team defeats a higher-positioned team. The rules are as follows:

  Matches are played alternately in odd-position rounds and in even-position rounds.

  In an odd-position round, teams in position 3 and 5 play against teams positioned immediately above them.

  In an even-position round, teams in position 2 and 4 play against teams positioned immediately above them.

  When a lower-positioned team defeats a higher-positioned team, the two teams switch positions after the round is completed.

19. Which one of the of following could be the order of teams, from position 1 through position 5 respectively, after exactly one round of even-position matches if no odd-position round has yet been played?

  (A) J , R, M, L, S
  (B) J, R, S, L, M
  (C) R, J, M, L, S
  (D) R, J, M, S, L
  (E) R, S, J, L, M

20. If exactly two rounds of matches have been played, beginning with an odd-position round, and if the lower-positioned teams have won every match in those two rounds, then each of the following must be true EXCEPT:

  (A) L is one position higher than J.
  (B) R is one position higher than L.
  (C) S is one position higher than R.
  (D) J is in position 4.
  (E) M is position 3.

21. Which one of the following could be true after exactly two rounds of matches have been played?

  (A) J has won two matches.
  (B) L has lost two matches.
  (C) R has won two matches.
  (D) L's only match was played against J.
  (E) M played against S in two matches.

22. If after exactly three rounds of matches M is in position 4, and J and L have won all of their matches, then which one of the following can be true?

  (A) J is in position 2.
  (B) J is in position 3.
  (C) L is in position 2.
  (D) R is in position 1.
  (E) S is in position 3.

23. If after exactly three rounds M has won three matches and the rankings of the other four teams relative to each other remain the same, then which one of the following must be in position 3?
 
  (A) J
  (B) L
  (C) M
  (D) R
  (E) S

24. If after exactly three rounds the teams, in order from first to fifth position, are R, J, L, S, and M, then which one of the following could be the order, from first to fifth position, of the teams after the second round?

  (A) J, R, M, S, L
  (B) J, L, S, M, R
  (C) R, J, S, L, M
  (D) R, L, M, S, J
  (E) R, M, L, S, J

上一页  [1] [2] [3] [4] 

责任编辑:wangpeng6151

文章搜索:
 相关文章
热点资讯
热门课程培训